Last visit was: 01 May 2024, 15:37 It is currently 01 May 2024, 15:37

Close
GMAT Club Daily Prep
Thank you for using the timer - this advanced tool can estimate your performance and suggest more practice questions. We have subscribed you to Daily Prep Questions via email.

Customized
for You

we will pick new questions that match your level based on your Timer History

Track
Your Progress

every week, we’ll send you an estimated GMAT score based on your performance

Practice
Pays

we will pick new questions that match your level based on your Timer History
Not interested in getting valuable practice questions and articles delivered to your email? No problem, unsubscribe here.
Close
Request Expert Reply
Confirm Cancel
SORT BY:
Date
Tags:
Show Tags
Hide Tags
CrackVerbal Representative
Joined: 22 Apr 2019
Posts: 100
Own Kudos [?]: 310 [13]
Given Kudos: 27
Send PM
Intern
Intern
Joined: 26 Oct 2018
Posts: 24
Own Kudos [?]: 32 [0]
Given Kudos: 30
Send PM
Current Student
Joined: 26 May 2019
Posts: 737
Own Kudos [?]: 263 [1]
Given Kudos: 84
Location: India
GMAT 1: 650 Q46 V34
GMAT 2: 720 Q49 V40
GPA: 2.58
WE:Consulting (Consulting)
Send PM
Director
Director
Joined: 14 Dec 2019
Posts: 829
Own Kudos [?]: 890 [0]
Given Kudos: 354
Location: Poland
Concentration: Entrepreneurship, Strategy
GMAT 1: 640 Q49 V27
GMAT 2: 660 Q49 V31
GMAT 3: 720 Q50 V38
GPA: 4
WE:Engineering (Consumer Electronics)
Send PM
Despite improvements in treatment for bronchitis.. [#permalink]
ravigupta2912 wrote:
I did get this right since negating B would absolutely destroy the conclusion and open doors to an alternate reason other than Nebulisers. But can someone explain E to me? I couldn't figure out what negating that would imply.

EDIT - Could it be that negating B would give "not shown a similar increase" and we are not really concerned with the rate of increase but with the cause of such increase (since the conclusion talks of cause while equalising the conditions which lead to Bronchitis)


ravigupta2912 :- I would usually not negate the choices that don't make sense at all.

For eg. E discusses about urban population - a point that the author has already thrown out of consideration as evident in the premise supporting the conclusion. Even if you negate that choice it wouldn't add anything to the argument since urban population is out of consideration for the cities under discussion. Moreover, E discusses about cities with heavy urban population - not a focus of this argument
Current Student
Joined: 26 May 2019
Posts: 737
Own Kudos [?]: 263 [0]
Given Kudos: 84
Location: India
GMAT 1: 650 Q46 V34
GMAT 2: 720 Q49 V40
GPA: 2.58
WE:Consulting (Consulting)
Send PM
Re: Despite improvements in treatment for bronchitis.. [#permalink]
Thanks shameekv1989. So important to read carefully. I hadn't noticed the "urban" angle but now that I re read it, it makes so much sense.

Thank you.

Posted from my mobile device
Director
Director
Joined: 11 Sep 2022
Posts: 500
Own Kudos [?]: 154 [0]
Given Kudos: 2
Location: India
Paras: Bhawsar
GMAT 1: 590 Q47 V24
GMAT 2: 580 Q49 V21
GMAT 3: 700 Q49 V35
GPA: 3.2
WE:Project Management (Other)
Send PM
Re: Despite improvements in treatment for bronchitis.. [#permalink]
(A) Urban pollution has not doubled in the past decade.
This option is not necessary for the argument's conclusion. The argument doesn't rely on whether urban pollution has doubled or not; instead, it focuses on the fact that there has been an increase in deaths from bronchitis even in cities with little or no urban pollution.

(B) Increased urban pollution, improved recording of bronchitis deaths, and the use of bronchial nebulizers are the only possible explanations of the increased death rate due to bronchitis.
This is the correct assumption. The argument dismisses the other two explanations (improved recording of deaths and increased urban pollution) and concludes that the use of bronchial nebulizers is the cause of the increased death rate. Therefore, it assumes that these three factors are the only plausible explanations, making (B) the necessary assumption.

(C) Bronchial nebulizers may cause side effects even when used according to the recommended instructions.
This statement, while possibly true, is not necessary for the argument's conclusion. The argument is concerned with whether nebulizer use is the primary cause of the increased death rate, not whether nebulizers have side effects.

(D) The use of bronchial nebulizers may aggravate other diseases that frequently occur among bronchitis sufferers.
Similar to (C), this statement, while possibly true, is not necessary for the argument's conclusion. The argument is focused on whether nebulizer use is the primary cause of the increased death rate, not whether it exacerbates other diseases.

(E) The death rate from bronchitis has shown a similar increase even in cities with heavy urban pollution.
This statement is interesting but not necessary for the argument's conclusion. The argument's focus is on the fact that the death rate from bronchitis has increased even in cities with little or no urban pollution, which suggests that urban pollution alone is not the explanation. This doesn't directly address the assumption that (B) makes, which is that there are only three possible explanations for the increased death rate.

In summary, option (B) is the correct assumption because it establishes that the three factors mentioned in the argument are the only plausible explanations for the increased death rate, thereby supporting the conclusion that nebulizer use is the cause.
GMAT Club Bot
Re: Despite improvements in treatment for bronchitis.. [#permalink]
Moderators:
GMAT Club Verbal Expert
6923 posts
GMAT Club Verbal Expert
238 posts
CR Forum Moderator
832 posts

Powered by phpBB © phpBB Group | Emoji artwork provided by EmojiOne